Sunteți pe pagina 1din 43

Chapter 4

Solution of Time-independent
Schrdinger Equation I
Bound States
A particle being confined by an external force to a finite spatial
region is said to be in a bound state.
Classical Mechanical Bound State
Example: Spring-mass system on a frictionless table
E=U+T
T=E-U0
EU
Quantum Mechanical Bound State
Example: A free light atom bound to a stationary heavy atom
-Classically speaking,
Atom 1 can only be in the
region x
a
<x<x
b
. Its
position x(t) is well known
at any time t. The total
energy E can take any
value, depending on the
initial setting.
-For Quantum Physics treatment, the particles position is unknown at
any time t. The probability density function |(x)|
2
is non-zero at the
classical forbidden region. The allowed total energy of the particle is
quantized.
Conceptual question 1
Which of the following statement(s) is/are true for a particle
in a quantum-mechanical bound state?
(i) The particle is confined to a finite region of space.
(ii) Zero probability of detecting the particle in the
classically forbidden region.
(iii) Energy of the particle is quantized.
Answer:
Only (i) and (iii) are true. Since the wave function can
penetrate into the classically forbidden region, there will be a
finite probability of detecting the particle in this region.
Solving the TISE for a piecewise-constant U
The time-independent Schrdinger equation (TISE):
If U(x) = constant in a region, this equation becomes:
where is a constant.
What is the form of the solution if C > 0 (E<U)?
How about the case for C < 0 (E>U)?
Conceptual question 2
Consider a particle with the potential energy as shown in
below figure.
(a) How many regions should the space be divided
according to the values of the potential energy?
(b) If the particle has energy U
0
< E < U
1
, classify each
region as classically allowed or classically forbidden.
Conceptual question 2 (cont.)
Answer:
(a) The space should be divided into the following regions:
Region I: x < x
1
Region II: x
1
< x < x
2
,
Region III: x > x
2
(b) Regions I and III are classically allowed. But region II is
classically forbidden.
Particle in a 1D box with infinite potential

U(X) ={
0, 0 x L
, x < 0 or x > L
For x<0 or x>L, U(x), thus:
The only possible solution is
i.e. the particle cannot be found in this region.

(x) = 0
For 0xL, U(x)=0,


2
2m
d
2
(x)
dx
2
= E(x)
(x) = Asinkx + Bcoskx, where k =
2mE

Boundary conditions: (x) continuous at x=0 and x=L



(x = 0) = A 0 + B 1= 0
B = 0
(x = L) = AsinkL = 0
k
n
L = n k
n
=
n
L
, n =1,2, 3,....

n
=
2
k
n
=
2L
n
=
or
h
2mE
n


2mE

=
n
L
E
n
=
n
2

2
2mL
2
, n =1,2, 3...
This implies the allowed energies of the particle are quantized.
For the particular quantum number n:
Normalization implies:

n
(x) = Asin
n
L
x







(x, t)
2
dx =
all space


n
(x)
2
dx

= A
2
sin
2
n
L
x






dx
0

=1
A =
2
L
i.e.,
n
=
2
L
sin
nx
L






, 0 x L
0, x < 0 or x > L

n
(x) = Asin
n
L
x






, n =1,2, 3 and 4

n
=
2L
n
-The lowest possible energy is called the ground
state energy, which corresponds to the n=1 ground state
wavelength
1
(x).
-The states with n>1 are called excited states.
-Notice that there is no solution for E=0 (i.e. n=0). A bound
quantum particle cannot be stationary. Otherwise, it will violate
the Heisenberg Uncertainty Principle .
-Expectation value of x.
which is independent of the quantum number n.


E
1
=

2

2
2mL
2


xp
x
= Lp
x
2

x = x*(x)(x)dx
0
L

= x
2
L
sin
2
nx
L
[
\
|

)
j
dx
0
L

=
2
L
x
1
2
1cos 2
nx
L
[
\
|

)
j
|
|
|
|
|
|
dx =
L
2
0
L

Conceptual question 3
How would the solution of the 1-D infinite potential well be
different if the width of the well is extended from L to (L +
x
0
) where x
0
> 0? How would the energies be different?
Conceptual question 3 (cont.)
Answer:
The form of the wave functions inside the well remains the
same. They still exist as stationary states described by the
same sinusoidal functions, except that in the expressions of
the observables, such as the quantized energies and the
expectation values, the parameter L be replaced by (L + x
0
).
For the quantized energies, they will be modified as
Particle in a box with finite potential (Finite potential well)
We only consider the case of E<U
0
Notice that Region II and III are classical forbidden regions.

U(X) ={
0, 0 x L
U
0
, x < 0 or x > L


d
2

dx
2
=
2m U(x) E
[ ]

2
(x)
For 0 x L, U(x) = 0 < E, implying a oscillatory solution, i.e.
(x) = Asin kx +
( )
, where k =
2mE

A and are constants


For x 0 or x L, U(x) =U
0
> E, implying exponential solution. i.e.,
(x) = Bexp -x
( )
+ Cexp x
( )
, where =
2m U
0
E
( )

2
C=0, due to diverge at x
In the classical forbidden region, (x)=Bexp(-x), which is non-
zero. This implies probability of finding the particle in the region is
not zero. This is called tunneling.
Penetration Depth
-The particle wave function is non-zero in the classical forbidden
region, and varies as ~ exp( - |x| ).
- is defined as the distance over which the wave function drops
by a fraction of 1/e, i.e.


=
1

=

2m U
0
E
( )
The negative KE
- For particle detected in the classical forbidden region, E<U(x)
and thus the KE is negative!!!
-The experiment that can have the detection of the particle in the
classical forbidden region MUST have the position uncertainty:
which corresponds to the uncertainty in KE of:
which is of the same magnitude of the negative KE.


x
so that by the Heisenberg Uncertainty Principle :
xp
p

x

= 2m U
0
E
( )

KE = p
2
/2m U
0
E
Not attempting to exactly obtain the solution, but only to
understand how the energy quantization is introduced.
Continuity of (x=L) implies:
Possible solutions can be find by plotting the graphs and finding
their intersections of:


kAsin kL +
( )
= Bexp L
( )

2mE

sin
2mE

L +






=
2m U
0
E
( )

B
A






exp
2m U
0
E
( )

L








sin
2mE

L +






=
B
A






U
0
E
1exp
2m U
0
E
( )

L










sin
2mE

+






vs E and
B
A






U
0
E
1exp
2m U
0
E
( )

L








vs E
Energy
Quantization
Finite vs Infinite well
Conceptual question 4
An electron is placed into the ground state of an infinite well
that has width L. Another electron is placed into the ground
state of a finite well that has a depth of U
0
and also has width
L as shown below. Compare the wavelengths of the two
electrons. Based on the ratio of the wavelengths, compare the
ground state energy of these two wells.
Conceptual question 4 (cont.)
Answer:
The probability of finding the electron outside of the infinite
well is zero, but the probability of finding the particle outside
of the finite well is non-zero. This effectively lengthens the
wavelength of the electron in the finite well as compared to
the infinite well. The momentum, and hence the kinetic
energy, is inversely related to the wavelength. So the finite
well has lower ground state energy.
Sketching wave functions
Consider the potential energy:
( a) Sket ch
4
( x) f or t hi s
potential and for V
0
= 0.
Compare your two wave
functions and explain the
differences between them.
(b)Is the energy E
4
for V
0
=
L greater or smaller than
the corresponding energy
E
4
for V
0
= 0? Why?
0
x

V
0


L
Answer:
(a) Note that:
1.
4
state has _4_ antinodes.
2. Wave function must go to zero at _x = 0_ and _x = L_.
3. Kinetic energy decreases_ as x L, so the
wavelength increases_ as x L.
(b) E
4
is greater for V
0
= L
Conceptual question 5
Below figure shows a solution of the Schrdinger equation.
(a) What is the value of the quantum number n? Explain how
you know.
(b)What is the probability of finding the particle at x < 0?
What is the probability of finding the particle at x > L?
Conceptual question 5 (cont.)
(d)At what value or values of x is the particle most likely to
be found?
(e) In what region(s) is (are) the de Broglie wavelength the
longest? In what region(s) is (are) the de Broglie
wavelength the shortest?
(f) In what region(s) would the particle move the fastest? In
what region(s) would the particle move the slowest?
(g)Now sketch a possible potential well in which the particle
is confined.
Conceptual question 5 (cont.)
Answer:
(a) n = 9 since the wave function has n nodes and (n - 1)
antinodes excluding the end.
(b) The probability of finding the particle at x < 0 and x > L
are both zero.
(c) The particle is most likely to be found at x = L/2.
(d) The de Broglie wavelength is the longest at the center
and the shortest at the end.
(e) The particle would move the fastest at the end and the
slowest at the center.
Conceptual question 5 (cont.)
Answer: (cont.)
(f)
Conceptual question 6
Consider a particle in the ground state of a finite well.
Describe the changes in its wave function and energy as the
walls are made progressively higher until essentially infinite?
Answer:
Its wave function will penetrate less into the classically
forbidden region, which in turn will reduce its wavelength
and thus slightly increase its energy.
Simple harmonic oscillator
Here, we consider the harmonic oscillator
potential:
A classical SHO
The harmonic oscillator potential is a good approximation
for:
(1) a vibrating diatomic molecule
(2) a system near the potential energy minimum
Quantum Simple Harmonic Oscillator


F = x = m
0
x, U(x) =
1
2
x
2
=
1
2
m
0
x
2
, where
0
=

m
TISE :

2
2m
d
2
(x)
dx
2
+
1
2
x
2
(x) = E(x)
Solution :
n
(x) =
1
2
n
n!
m
0







1/ 4
e

m
0
x
2
2
H
n
m
0

x






,
n = 0,1,2, 3,.....


H
n
(x) = 1
( )
n
exp x
2
( )
d
n
dx
n
exp x
2
( ) [ ]
is called the Hermite polynomial.
The corresponding energies :
E
n
= n +
1
2





0
Allowed energies and wave functions for the SHO
Conceptual question 7
In the harmonic oscillator wave functions, there is a variation
in wavelength from the middle to the extremes of the
classically allowed region, most noticeable in the higher-n
functions. Why does it vary as it does?
Answer:
It tends to get longer near the extreme edges. The reason is
that the kinetic energy is low there, so the momentum is
small, corresponding to a large wavelength.
Conceptual question 8
A particular laser that emits at a wavelength = 2.7 m
operates by exciting hydrogen fluoride (HF) molecules
between their ground and 1
st
excited states. Estimate the
ground state energy of the HF molecule vibrations.
Answer:
The energy of the photon
And
By conservation of energy,
So the ground state energy is
Operators and Expectation Values
The expectation of an observable Q is given as:
e.g.


Momentum Operator :

p = i

x
Position Operator :

x = x
Energy Operator :

E = i

t

Q = Q = *(x, t)
all space


Q (x, t)dx


KE =
p
2
2m
KE
^
=
1
2m

p
2
=
1
2m
i

x






i

x






=

2
2m

2
x
2
For a time-independent potential U(x), energy conservation implies:
-TDSE


KE +U(x) = E
KE
^
+U(

x ) =

E

i
2
2m
(x, t) +U(x)(x, t) = i
(x, t)
t

S-ar putea să vă placă și